subject
Mathematics, 13.07.2019 21:00 gautemalaismylife

Asap a box plot was made to represent the number of matches won by 14 participants in a tennis tournament. the box plot had the box shifted to the left so that the right tail was much longer than the left tail. based on the box plot, which conclusion is correct? the mean and median of matches won are equal. the mean of matches won is less than the median of matches won. most of the participants won many matches, but some participants won very few matches compared to the others. most of the participants won very few matches, but some participants won many matches compared to the others.

ansver
Answers: 1

Other questions on the subject: Mathematics

image
Mathematics, 21.06.2019 19:00, amayareyes101
What are the solutions of the equation? z^2 + 11z + 24 = 0 a. 8, -3 b. 8, 3 c. -8, -3 d. -8, 3
Answers: 2
image
Mathematics, 21.06.2019 19:30, Davon21
If the ratio of sum of the first m and n terms of an ap is m2 : n2 , show that the ratio of its mth and nth terms is (2m − 1) : (2n − 1).
Answers: 3
image
Mathematics, 22.06.2019 04:00, jamiecoolgal8697
What is the answer to this equation? and if you can, find a way to show your work.
Answers: 2
image
Mathematics, 22.06.2019 05:30, bartlettcs9817
Aletter in the word electric is chosen at random. complete each sentence. there are possible outcomes. the theoretical probability that a vowel will not be chosen is the ratio . the theoretical probability that the letter n will be chosen is .
Answers: 2
You know the right answer?
Asap a box plot was made to represent the number of matches won by 14 participants in a tennis tou...

Questions in other subjects: